Discuss the market risk premium


Problem 1. Risk- Free Rate

A stock has an expected return of 14 percent, a beta of 1.70, and the expected return on the market is 10 percent. What must the risk-free rate be?
Risk-Free Rate = ?

Problem 2. Market Risk Premium

A stock has a beta of .8 and an expected return of 13 percent. If the risk-free rate is 4.5 percent, what is the market risk premium?
Market risk premium = ?

Please show the calculation along with the answer.

Solution Preview :

Prepared by a verified Expert
Finance Basics: Discuss the market risk premium
Reference No:- TGS02037606

Now Priced at $20 (50% Discount)

Recommended (94%)

Rated (4.6/5)